4
$\begingroup$

Let $H,M\geq 1$ and let $h_0$ and $m_0$ be fixed integers with $(h_0,m_0)\in [H,2H]\times[M,2M]$. Let $\alpha$ be a positive real number. I'm trying to find an upper found for the number of integer pairs $(h,m)\in [H,2H]\times[M,2M]$ such that $$ \tag{1} m h^\alpha = m_0 h_0^\alpha. $$ If $\alpha$ is rational, say $\alpha = \frac{p}{q}$, then $m^q h^p = m_0^q h_0^p$, and so the number of solutions is at most the number of divisors of $m^q h^p$, which is $O((HM)^\varepsilon)$ for any $\epsilon > 0$, the implied constant depending on both $\alpha$ and $\varepsilon$.

When $\alpha$ is irrational, I expect that the same bound should hold, but I'm not sure how to show this. Thus my question,

Is the number of solutions to (1) bounded by $O((HM)^\varepsilon)$ when $\alpha$ is irrational?

For context, the application I have in mind is a bound for certain exponential sums in the spirit of the classical paper "Exponential sums with monomials" by Fouvry and Iwaniec.

Any feedback and references are most appreciated.

$\endgroup$
6
  • $\begingroup$ If $\alpha$ is irrational, doesn't it imply $(m,h)=(m_0,h_0)$? $\endgroup$ Commented Dec 6, 2023 at 2:47
  • 1
    $\begingroup$ No, because $\alpha$ could be a quotient of logarithms, say $\alpha = \frac{\log(m/m_0)}{\log(h_0/h)}$. One then needs to show that this equation doesn't have too many solutions in $(h_0,m_0)$, which is essentially an equivalent problem. $\endgroup$ Commented Dec 6, 2023 at 3:36
  • $\begingroup$ Given two solutions $(h_1, m_1), (h_2, m_2)$, you have $\log(\frac{m_1}{m_0}) \log(\frac{h_2}{h_0}) = \log(\frac{m_2}{m_0})\log(\frac{h_1}{h_0})$. There might be something simple this implies which I'm missing, but assuming a particular case of Schanuel's conjecture this implies that those logarithms are linearly dependent, which seems useful. $\endgroup$ Commented Dec 6, 2023 at 4:37
  • $\begingroup$ Ah, okay. Thanks. $\endgroup$ Commented Dec 6, 2023 at 5:27
  • 1
    $\begingroup$ check Lucia's comment here mathoverflow.net/q/332532/4312 $\endgroup$ Commented Dec 6, 2023 at 6:45

0

You must log in to answer this question.

Start asking to get answers

Find the answer to your question by asking.

Ask question

Explore related questions

See similar questions with these tags.